User Avatar
sammariscal07533
Joined
Apr 2025
Subscription
Free
User Avatar
sammariscal07533
Monday, May 05 2025

From my own perspective and experience, but printing off LR pdfs is not very helpful. I am one of those persons that needs to see text on paper, so I understand that the paper approach can seem helpful for a variety of reasons. But when it comes to this, yeah, there are better ways to studying than printing PDFs. Probably the worst approach, honestly.

1
PrepTests ·
PT146.S2.Q13
User Avatar
sammariscal07533
Wednesday, Apr 30 2025

The argument is not simply "that the campaign caused the flu to decrease." The conclusion is that the campaign was successful because people heeded the advice." So the assumption, perhaps the necessary assumption, the "loop" here is that because people heeded the advice, the campaign was successful. (A) Strengthens by showing that indeed people heeded the advice, thus strengthening the conclusion. If people washed their hands for another arbitrary reason then that would weaken the argument, which is what (C) does. This would mean that there was another alternative to the campaign.

1
PrepTests ·
PT146.S2.Q13
User Avatar
sammariscal07533
Wednesday, Apr 30 2025

If anything (C) weakens the argument by showing alternative cause .

2
PrepTests ·
PT117.S1.P4.Q23
User Avatar
sammariscal07533
Monday, Apr 21 2025

In this passage I imagined the author being a certain builder in a blue suit and long red tie calling the Modern Movement losers and idiots, total morons, for making the worst buildings in the history of buildings, while he built the biggest most beautiful buildings. Yet in question #23, being "dismissive", "dictating", and having "confirmation bias", is only being unimpressed? Maybe I'm just not familiar with the way people in the building industry speak, but this passage seemed a lot more hyperbolic and exasperating.

3
PrepTests ·
PT117.S1.P3.Q20
User Avatar
sammariscal07533
Monday, Apr 21 2025

I have no idea how I only missed the last question. I was half asleep at 3am when I drilled this passage. I only picked my answer because I wanted to end the drill.

0
PrepTests ·
PT140.S2.Q6
User Avatar
sammariscal07533
Sunday, Apr 06 2025

Yeah, I'm not sure where you're getting that people are caught the first time.

0
PrepTests ·
PT129.S2.Q10
User Avatar
sammariscal07533
Sunday, Apr 06 2025

I think you meant "I don't like B" . Farm animals are a superset and cows are a subset of farm animals. So in these occasions you should ask yourself whether the referential is talking about the subset or the superset. Here, we have to assume that cows are farm animals, and that by "farm animals" we don't mean chickens.

0
User Avatar
sammariscal07533
Friday, Apr 04 2025

LeFLOP

But to the question, it depends. Timed, I read the QStem. Blind Review, I read the stimmy.

In BR, the point is to try to bend and flex the argument in any direction you want. So it's not simply about answering the question, but rather using the stimmy to answer many potential questions. For example, RRE, Necessary Assumptions, Weaken, Strengthen, Sufficient Assumption, Descriptive Weakening, Must Be True (MBT conclusions are NA premises), Evaluate, and possibly others, rely on finding an unstated assumption that at times must be true or is true to some degree. So reading the Qstem first is such a small part of the process.

0
PrepTests ·
PT114.S3.P3.Q15
User Avatar
sammariscal07533
Monday, Mar 31 2025

LOL at the "I'm offended" crowd of several years ago. Hoping they've grown out of their pink hair phase.

1
PrepTests ·
PT120.S3.Q14
User Avatar
sammariscal07533
Tuesday, Mar 11 2025

wat

0
PrepTests ·
PT105.S2.Q2
User Avatar
sammariscal07533
Sunday, Feb 02 2025

That Fauci ouchie

0
PrepTests ·
PT111.S3.Q11
User Avatar
sammariscal07533
Monday, Jan 06 2025

The implicit promise IS referring to the teacher and the student's conversation

0
PrepTests ·
PT106.S3.Q2
User Avatar
sammariscal07533
Friday, Dec 20 2024

The author interprets or actually chooses to ignore "a number" and simply creates the conditional GW, then Criminals. It then assumes Grandville's Planning Committee should NOT have any criminals on it, or the Contrapositive, IF criminals, then NOT GPC. To then conclude that IF GPC, then NOT GW, which then is the Contrapositive of the linked conditional.

GW -> C

C -> /GPC

GW -> /GPC (GPC -> /GW)

The author thinks that its argument is valid (otherwise why make it), but makes two faulty assumptions: One, that "a number" of Granville's wealthiest are criminals actually means ALL of Grandville's wealthiest are criminals (GW -> C). The second faulty assumption is that "Grandville's Planning Committee should ONLY consist of persons with ethics beyond reproach" actually means "Grandville's Planning Committee should ONLY consist of NO criminals."

The problem with this second conditional is not an ethics question about whether criminals do or do not have ethics beyond reproach. Rather, it's missing a conditional that tells us so. Meaning, there is a missing conditional that says: "IF Criminals, then NOT ethics beyond reproach."

I just want to make this clear because had this been a sufficient or necessary assumption question, this non existent conditional would have been a major player. Here, it certainly is a necessary assumption in order for the author to make his conclusion, but it's necessary based on the author ignoring "a number" and simply diving straight into ALL and ONLY conditional language throughout the stimmy.

0
PrepTests ·
PT106.S3.Q2
User Avatar
sammariscal07533
Friday, Dec 20 2024

Hi E,

I also got this question wrong, but I chose (A). I wanted to try to help you understand why (C) is wrong, just based on my personal studying so far. The LSAT is very picky about "intentions" and "motives". Meaning, that each are very hard to prove. In my time studying this exam, most answers that are worded to assume intentions and motives are wrong 99% of the time. Unless the stimulus explicitly gives you a motive or an intention, the answer is wrong.

One thing you can correct is your assumption of the LSAT. When you say that "I thought that if the criteria is ethics, the law doesn't always align to ethics." Where in the argument does it say this? See, you're brining in your own assumptions, your own motives and intentions, which the LSAT never made. Here the LSAT tricked you and baited you to assume your own world view into the argument. Think of each argument as its own little world. Each question requires you treat the stimulus as its own world and make a choice ONLY on what exists in that world.

There are certain patterns each "world" has in common. For example, many paradoxes in this world require a causal solution. One world may actually present you with a causal solution, but you being careful about it notice that there are things or facts in this world that point to alternative reasons or causes not presented to you. So to improve, you need to focus on each passage and understand it on its own terms.

Additionally, you're not necessarily trying to weaken this argument. Which I think is why you made your assumption. You possibly wanted to find a way that this argument could be weakened, so you assumed something outside of the argument itself that would perhaps divert the logic into another conclusion. Well, actually, in these descriptive weakening questions you are attempting to find an answer that describes the wrong method of reasoning. So the author wrongly reasons something, and your job is to find the answer that describes the wrong method of reasoning the author made. Another reason to avoid imposing your own assumptions.

Hope this helps! :)

0
PrepTests ·
PT146.S4.P2.Q11
User Avatar
sammariscal07533
Thursday, Dec 19 2024

My understanding of the question stem, seemingly yours, was that the author implied that Taruskin must end in a "Freudian slip".

But just to clarify, the author is not suggesting that Taruskin SHOULD hold this view, rather that Taruskin, according to author's arguments in paragraphs 3 and 4, MUST hold this view presented in paragraph 5. In other words, in the author's view, Taruskin's assumptions ONLY leads us to one conclusion, that elites secretly and intentionally sponsored art they hated.

So I think you almost had it, except that in my opinion, you misunderstood the question stem. The question stem is not asking you to describe Taruskin's view point, but rather what the passage, I.e., the author of the passage, argues about Taruskin's view point, which is that we follow Taruskin's view point to its logical conclusion, it MUST be a "Freudian slip."

0
PrepTests ·
PT145.S1.P3.Q14
User Avatar
sammariscal07533
Wednesday, Dec 18 2024

While reading I kept thinking that sooner rather than later the tone of this passage would be "I'm offended."

0
PrepTests ·
PT123.S2.Q14
User Avatar
sammariscal07533
Sunday, Nov 17 2024

There is a difference between the microwaves themselves and the heat the microwaves create. Picture a glass in the microwave. Microwaves are hitting the glass as a whole. The argument claims or concludes that the microwaves themselves are killing the bacteria. So you have these little waves directly hitting and killing the bacteria, and ONLY the waves themselves are killing the bacteria (no heat). Such a certain cause is easily weakened just by simply saying that the microwaves actually cause heat. Once you bring heat back into it, the MC is weakened because that's what the author is intentionally excluding.

What makes this question so hard for us is that we cannot think of microwaves as just waves in themselves hitting or zapping bacteria directly. Think of microwaves as lasers. The lasers are killing the bacteria, says the author. not the heat that the lasers could create.

1
PrepTests ·
PT116.S3.Q19
User Avatar
sammariscal07533
Tuesday, Nov 05 2024

In these cases I always try to remind myself that the goal of a Weaken question is not to crush the author's argument, but simply to cause the author to question their conclusion.

1
PrepTests ·
PT113.S4.Q2
User Avatar
sammariscal07533
Sunday, Nov 03 2024

Yes, the researchers explicitly mention that they stimulated human VNO cells, but the subjects "reported" that they smelled something. The assumption here is that ONLY VNO cells caused the smell. Therefore, the argument concludes that VNO is probably a sensory organ. What's the weakness? The premise is not what the researches did, it's what the subjects "reported." (A) Allows for the premise and the main conclusion to stand while targeting the assumption that ONLY VNO cells cause smell functions in humans. There is a huge gap between the test and the results. The testers concluded something based on what the subjects reported. Anytime this happens on an LSAT question it's always sus. The answer is not challenging the method, but rather the conclusion based on what the testers reported.

2
PrepTests ·
PT108.S2.Q3
User Avatar
sammariscal07533
Monday, Oct 21 2024

I interpreted this argument as a Correlation - Causation argument. I didn't originally pick (A) because I thought it directly countered the premises. The first line says "Polls negatively influence voters." I thought (A) was like, "nah, it doesn't." I picked (C). But then I realized that (C) strengthens the argument by showing one example of "bad influence." All other answers sucked. So I went back to (A) and realized that we could allow the cause (MC) to stand, but (A) shows that while there is a cause, it shows that the assumed outcome of the cause does not occur. After all, the author does assume that polling is causing the bad influence. Allowing the cause to stand but negating the assumed affect is one way to weaken.

0
PrepTests ·
PT104.S3.P3.Q20
User Avatar
sammariscal07533
Friday, Oct 18 2024

I think one way to find the correct answer is to contrast (B) and (D). Answer (B) strengthens Papi's argument (pigeons use smell to go home) by showing that when smell is partially blocked, the pigeons partially find their way home (more slowly). In other words, smell causes direction. Less smell, less cause of direction. Answer (D) then sows that when their is no smell (removing the cause) then the affect is removed, too. The smell is removed by transporting them well beyond the range of smell. So taking that as true, that the smell is removed (the cause is removed) then reasonably so the affect of them flying home is removed. Because this is a WKN question, the reasoning used in both (B) and (D) is causal. (D) WKNs by removing the cause and thus removing the affect.

0
PrepTests ·
PT104.S3.P3.Q16
User Avatar
sammariscal07533
Friday, Oct 18 2024

Honestly, for a name like Papi and what he did to those dang birds, I'd kick him while he's down too.

3
PrepTests ·
PT108.S3.Q6
User Avatar
sammariscal07533
Wednesday, Oct 16 2024

#HELP

I know that this is sort of a silly assumption, but answer (C) assumes that the fish in the pond either remained constant or actually increased. If there were more fishermen fishing and we had a constant number of fish, then the fish population would probably go down, at the very least have a 1:1 ratio, versus if we had less fisherman perhaps we had a 1:10 ratio. I was so caught up in trying to explain to myself that just because we have a greater number of fishermen does not mean we have a greater number of fish caught. But yes, the "per fisherman" would have helped clear this up, but I don't trust the LSAT to give in that easily. Maybe I'm reasoning this problem as a # to % argument?

5
PrepTests ·
PT101.S4.P2.Q11
User Avatar
sammariscal07533
Thursday, Oct 10 2024

It's not necessarily more difficult because "standing" for ancient graves means that the ancient artifacts that were once perceived as private property actually become communal property. So if a tribe has standing in an ancient grave case, then it likely means that the tribe has communal property rights over the artifacts, as opposed to someone else who claimed the artifacts were personal property, as implied in paragraph 3.

0
PrepTests ·
PT101.S4.P2.Q11
User Avatar
sammariscal07533
Thursday, Oct 10 2024

Yeah, both (A) and (B) are wrong for similar reasons in that, to your point, once standing is established, it can help and does not necessarily have to be more difficult to resolve. Mostly, because standing for ancient graves means that communal property supersedes private property, as implied in paragraph 3. Understanding paragraph 2 and 3 is really crucial to this question. Once standing is established, it is not necessarily difficult to resolve because communal property supersedes private property. This quickly eliminates answers (A) and (B).

1

Confirm action

Are you sure?